Find the number of $k$ satisfying $frac{n}{3} - 1 9$.Inequlity $frac{2}{frac{a}{x}+frac{b}{y}}leq ax+by$ for...

Why not use SQL instead of GraphQL?

How does strength of boric acid solution increase in presence of salicylic acid?

How can bays and straits be determined in a procedurally generated map?

What is the word for reserving something for yourself before others do?

How do I create uniquely male characters?

How can I make my BBEG immortal short of making them a Lich or Vampire?

What's the output of a record cartridge playing an out-of-speed record

Can an x86 CPU running in real mode be considered to be basically an 8086 CPU?

Do VLANs within a subnet need to have their own subnet for router on a stick?

What defenses are there against being summoned by the Gate spell?

Mathematical cryptic clues

Arthur Somervell: 1000 Exercises - Meaning of this notation

In Japanese, what’s the difference between “Tonari ni” (となりに) and “Tsugi” (つぎ)? When would you use one over the other?

Languages that we cannot (dis)prove to be Context-Free

Theorems that impeded progress

How is the claim "I am in New York only if I am in America" the same as "If I am in New York, then I am in America?

Is it legal for company to use my work email to pretend I still work there?

What do the dots in this tr command do: tr .............A-Z A-ZA-Z <<< "JVPQBOV" (with 13 dots)

How much RAM could one put in a typical 80386 setup?

Why "Having chlorophyll without photosynthesis is actually very dangerous" and "like living with a bomb"?

Is this a crack on the carbon frame?

Python: next in for loop

"to be prejudice towards/against someone" vs "to be prejudiced against/towards someone"

The Two and the One



Find the number of $k$ satisfying $frac{n}{3} - 1 9$.


Inequlity $frac{2}{frac{a}{x}+frac{b}{y}}leq ax+by$ for $a+b=1$Find the number of integer solutions to $x_1+x_2+x_3+x_4= 30$ where $0leq x_n <10$ for $1leq n leq 4$Alternate approach to show $|sum_{i leq n} frac{mu(i)}{i}| leq 1$?Prove $| sum_{i leq n} frac{mu(i)}{i} | leq 1$Let $x$ be a real number. Prove the existence of a unique integer $a$ such that $a leq x < a+1$Prove that the number of fractions $a/b$ in lowest terms with $0<a/b leq 1$ and $bleq n$ is $phi(1) + … + phi(n)$.Prove that the nth prime number $p_n$ satisfies $p_nleq 2^{2n-1}$Prove that for every prime number $|{1leq x leq p^2 : p^2 | (x^{p-1}-1) }|=p-1$proof that $frac{m^a}{2}+frac{m}{2}-1$ is not a prime numberHow to find all the solutions for $sumlimits_{i=1}^{n} frac{1}{2^{k_i}}= 1$ for $k_iin Bbb{N}$ and $n$ a fixed positive integer?













1












$begingroup$


Show that the number of $k in mathbb{N}$ satisfying $frac{n}{3} - 1 < k leq frac{n}{2}$ for $n in mathbb{N}, n> 9$ is greater than $frac{n}{6}$.



I looked at the distance $|frac{n}{3} - 1 - (frac{n}{2})| = 1 + frac{n}{6}$ which is $> frac{n}{6}.$ Is this correct or am I missing a proper justification? I'm not sure how to use the $n > 9$ fact either. Thank you.










share|cite|improve this question









$endgroup$












  • $begingroup$
    You need to be careful with using the distance, as you did, in terms of the boundary conditions with the required inequalities and the value of $k$ being an integer. Although the total is $gt frac{n}{6}$, it's possible for there to be fewer than that integers which satisfy the inequalities. Regarding using $n gt 9$, as I show in my answer, doesn't seem to really be needed, so I'm not sure why it's provided. As I state in my answer, perhaps there's another required condition that is not included here.
    $endgroup$
    – John Omielan
    Mar 20 at 2:37










  • $begingroup$
    There were not any additional conditions provided so I was a bit dumbfounded myself. Thank you for your response, I understand now.
    $endgroup$
    – Darkdub
    Mar 20 at 9:33
















1












$begingroup$


Show that the number of $k in mathbb{N}$ satisfying $frac{n}{3} - 1 < k leq frac{n}{2}$ for $n in mathbb{N}, n> 9$ is greater than $frac{n}{6}$.



I looked at the distance $|frac{n}{3} - 1 - (frac{n}{2})| = 1 + frac{n}{6}$ which is $> frac{n}{6}.$ Is this correct or am I missing a proper justification? I'm not sure how to use the $n > 9$ fact either. Thank you.










share|cite|improve this question









$endgroup$












  • $begingroup$
    You need to be careful with using the distance, as you did, in terms of the boundary conditions with the required inequalities and the value of $k$ being an integer. Although the total is $gt frac{n}{6}$, it's possible for there to be fewer than that integers which satisfy the inequalities. Regarding using $n gt 9$, as I show in my answer, doesn't seem to really be needed, so I'm not sure why it's provided. As I state in my answer, perhaps there's another required condition that is not included here.
    $endgroup$
    – John Omielan
    Mar 20 at 2:37










  • $begingroup$
    There were not any additional conditions provided so I was a bit dumbfounded myself. Thank you for your response, I understand now.
    $endgroup$
    – Darkdub
    Mar 20 at 9:33














1












1








1





$begingroup$


Show that the number of $k in mathbb{N}$ satisfying $frac{n}{3} - 1 < k leq frac{n}{2}$ for $n in mathbb{N}, n> 9$ is greater than $frac{n}{6}$.



I looked at the distance $|frac{n}{3} - 1 - (frac{n}{2})| = 1 + frac{n}{6}$ which is $> frac{n}{6}.$ Is this correct or am I missing a proper justification? I'm not sure how to use the $n > 9$ fact either. Thank you.










share|cite|improve this question









$endgroup$




Show that the number of $k in mathbb{N}$ satisfying $frac{n}{3} - 1 < k leq frac{n}{2}$ for $n in mathbb{N}, n> 9$ is greater than $frac{n}{6}$.



I looked at the distance $|frac{n}{3} - 1 - (frac{n}{2})| = 1 + frac{n}{6}$ which is $> frac{n}{6}.$ Is this correct or am I missing a proper justification? I'm not sure how to use the $n > 9$ fact either. Thank you.







elementary-number-theory






share|cite|improve this question













share|cite|improve this question











share|cite|improve this question




share|cite|improve this question










asked Mar 20 at 0:39









DarkdubDarkdub

11516




11516












  • $begingroup$
    You need to be careful with using the distance, as you did, in terms of the boundary conditions with the required inequalities and the value of $k$ being an integer. Although the total is $gt frac{n}{6}$, it's possible for there to be fewer than that integers which satisfy the inequalities. Regarding using $n gt 9$, as I show in my answer, doesn't seem to really be needed, so I'm not sure why it's provided. As I state in my answer, perhaps there's another required condition that is not included here.
    $endgroup$
    – John Omielan
    Mar 20 at 2:37










  • $begingroup$
    There were not any additional conditions provided so I was a bit dumbfounded myself. Thank you for your response, I understand now.
    $endgroup$
    – Darkdub
    Mar 20 at 9:33


















  • $begingroup$
    You need to be careful with using the distance, as you did, in terms of the boundary conditions with the required inequalities and the value of $k$ being an integer. Although the total is $gt frac{n}{6}$, it's possible for there to be fewer than that integers which satisfy the inequalities. Regarding using $n gt 9$, as I show in my answer, doesn't seem to really be needed, so I'm not sure why it's provided. As I state in my answer, perhaps there's another required condition that is not included here.
    $endgroup$
    – John Omielan
    Mar 20 at 2:37










  • $begingroup$
    There were not any additional conditions provided so I was a bit dumbfounded myself. Thank you for your response, I understand now.
    $endgroup$
    – Darkdub
    Mar 20 at 9:33
















$begingroup$
You need to be careful with using the distance, as you did, in terms of the boundary conditions with the required inequalities and the value of $k$ being an integer. Although the total is $gt frac{n}{6}$, it's possible for there to be fewer than that integers which satisfy the inequalities. Regarding using $n gt 9$, as I show in my answer, doesn't seem to really be needed, so I'm not sure why it's provided. As I state in my answer, perhaps there's another required condition that is not included here.
$endgroup$
– John Omielan
Mar 20 at 2:37




$begingroup$
You need to be careful with using the distance, as you did, in terms of the boundary conditions with the required inequalities and the value of $k$ being an integer. Although the total is $gt frac{n}{6}$, it's possible for there to be fewer than that integers which satisfy the inequalities. Regarding using $n gt 9$, as I show in my answer, doesn't seem to really be needed, so I'm not sure why it's provided. As I state in my answer, perhaps there's another required condition that is not included here.
$endgroup$
– John Omielan
Mar 20 at 2:37












$begingroup$
There were not any additional conditions provided so I was a bit dumbfounded myself. Thank you for your response, I understand now.
$endgroup$
– Darkdub
Mar 20 at 9:33




$begingroup$
There were not any additional conditions provided so I was a bit dumbfounded myself. Thank you for your response, I understand now.
$endgroup$
– Darkdub
Mar 20 at 9:33










1 Answer
1






active

oldest

votes


















1












$begingroup$

Let $n = 6m + r$ for some non-negative integer $m$ and integer $0 le r le 5$. Thus, the question asks to prove there are more than $frac{n}{6} = m + frac{r}{6} lt m + 1$ values of $k$, i.e., that there are at least $m + 1$ such values.



The inequality becomes



$$2m + frac{r}{3} - 1 lt k le 3m + frac{r}{2} tag{1}label{eq1}$$



In general for large divisors, you would normally look at various spans of values of $r$. However, since there aren't too many here, you can instead fairly easily manually check the results of the $6$ possible values of $r$ to get:



$$r = 0 ; Rightarrow ; 2m - 1 lt k le 3m tag{2}label{eq2}$$
$$r = 1 ; Rightarrow ; 2m - frac{2}{3} lt k le 3m + frac{1}{2}tag{3}label{eq3}$$
$$r = 2 ; Rightarrow ; 2m - frac{1}{3} lt k le 3m + 1 tag{4}label{eq4}$$
$$r = 3 ; Rightarrow ; 2m lt k le 3m + frac{3}{2} tag{5}label{eq5}$$
$$r = 4 ; Rightarrow ; 2m + frac{1}{3} lt k le 3m + 2 tag{6}label{eq6}$$
$$r = 5 ; Rightarrow ; 2m + frac{2}{3} lt k le 3m + frac{5}{2} tag{7}label{eq7}$$



With eqref{eq2} and eqref{eq3}, $k$ goes from $2m$ to $3m$ inclusive, so there are $m + 1$ values which work. Similarly with eqref{eq4}, $k$ goes from $2m$ to $3m + 1$, so there are actually $m + 2$ values which work. With eqref{eq5}, $k$ goes from $2m + 1$ to $3m + 1$, for $m + 1$ values. Finally, eqref{eq6} & eqref{eq7} give $k$ going from $2m + 1$ to $3m + 2$, for $m + 2$ values.



As such, this works for all $n in mathbb{N}$, including for $n gt 9$. I'm not sure why this restriction is used in the question. Are there perhaps any other unstated conditions?






share|cite|improve this answer











$endgroup$














    Your Answer





    StackExchange.ifUsing("editor", function () {
    return StackExchange.using("mathjaxEditing", function () {
    StackExchange.MarkdownEditor.creationCallbacks.add(function (editor, postfix) {
    StackExchange.mathjaxEditing.prepareWmdForMathJax(editor, postfix, [["$", "$"], ["\\(","\\)"]]);
    });
    });
    }, "mathjax-editing");

    StackExchange.ready(function() {
    var channelOptions = {
    tags: "".split(" "),
    id: "69"
    };
    initTagRenderer("".split(" "), "".split(" "), channelOptions);

    StackExchange.using("externalEditor", function() {
    // Have to fire editor after snippets, if snippets enabled
    if (StackExchange.settings.snippets.snippetsEnabled) {
    StackExchange.using("snippets", function() {
    createEditor();
    });
    }
    else {
    createEditor();
    }
    });

    function createEditor() {
    StackExchange.prepareEditor({
    heartbeatType: 'answer',
    autoActivateHeartbeat: false,
    convertImagesToLinks: true,
    noModals: true,
    showLowRepImageUploadWarning: true,
    reputationToPostImages: 10,
    bindNavPrevention: true,
    postfix: "",
    imageUploader: {
    brandingHtml: "Powered by u003ca class="icon-imgur-white" href="https://imgur.com/"u003eu003c/au003e",
    contentPolicyHtml: "User contributions licensed under u003ca href="https://creativecommons.org/licenses/by-sa/3.0/"u003ecc by-sa 3.0 with attribution requiredu003c/au003e u003ca href="https://stackoverflow.com/legal/content-policy"u003e(content policy)u003c/au003e",
    allowUrls: true
    },
    noCode: true, onDemand: true,
    discardSelector: ".discard-answer"
    ,immediatelyShowMarkdownHelp:true
    });


    }
    });














    draft saved

    draft discarded


















    StackExchange.ready(
    function () {
    StackExchange.openid.initPostLogin('.new-post-login', 'https%3a%2f%2fmath.stackexchange.com%2fquestions%2f3154858%2ffind-the-number-of-k-satisfying-fracn3-1-k-leq-fracn2-for-n%23new-answer', 'question_page');
    }
    );

    Post as a guest















    Required, but never shown

























    1 Answer
    1






    active

    oldest

    votes








    1 Answer
    1






    active

    oldest

    votes









    active

    oldest

    votes






    active

    oldest

    votes









    1












    $begingroup$

    Let $n = 6m + r$ for some non-negative integer $m$ and integer $0 le r le 5$. Thus, the question asks to prove there are more than $frac{n}{6} = m + frac{r}{6} lt m + 1$ values of $k$, i.e., that there are at least $m + 1$ such values.



    The inequality becomes



    $$2m + frac{r}{3} - 1 lt k le 3m + frac{r}{2} tag{1}label{eq1}$$



    In general for large divisors, you would normally look at various spans of values of $r$. However, since there aren't too many here, you can instead fairly easily manually check the results of the $6$ possible values of $r$ to get:



    $$r = 0 ; Rightarrow ; 2m - 1 lt k le 3m tag{2}label{eq2}$$
    $$r = 1 ; Rightarrow ; 2m - frac{2}{3} lt k le 3m + frac{1}{2}tag{3}label{eq3}$$
    $$r = 2 ; Rightarrow ; 2m - frac{1}{3} lt k le 3m + 1 tag{4}label{eq4}$$
    $$r = 3 ; Rightarrow ; 2m lt k le 3m + frac{3}{2} tag{5}label{eq5}$$
    $$r = 4 ; Rightarrow ; 2m + frac{1}{3} lt k le 3m + 2 tag{6}label{eq6}$$
    $$r = 5 ; Rightarrow ; 2m + frac{2}{3} lt k le 3m + frac{5}{2} tag{7}label{eq7}$$



    With eqref{eq2} and eqref{eq3}, $k$ goes from $2m$ to $3m$ inclusive, so there are $m + 1$ values which work. Similarly with eqref{eq4}, $k$ goes from $2m$ to $3m + 1$, so there are actually $m + 2$ values which work. With eqref{eq5}, $k$ goes from $2m + 1$ to $3m + 1$, for $m + 1$ values. Finally, eqref{eq6} & eqref{eq7} give $k$ going from $2m + 1$ to $3m + 2$, for $m + 2$ values.



    As such, this works for all $n in mathbb{N}$, including for $n gt 9$. I'm not sure why this restriction is used in the question. Are there perhaps any other unstated conditions?






    share|cite|improve this answer











    $endgroup$


















      1












      $begingroup$

      Let $n = 6m + r$ for some non-negative integer $m$ and integer $0 le r le 5$. Thus, the question asks to prove there are more than $frac{n}{6} = m + frac{r}{6} lt m + 1$ values of $k$, i.e., that there are at least $m + 1$ such values.



      The inequality becomes



      $$2m + frac{r}{3} - 1 lt k le 3m + frac{r}{2} tag{1}label{eq1}$$



      In general for large divisors, you would normally look at various spans of values of $r$. However, since there aren't too many here, you can instead fairly easily manually check the results of the $6$ possible values of $r$ to get:



      $$r = 0 ; Rightarrow ; 2m - 1 lt k le 3m tag{2}label{eq2}$$
      $$r = 1 ; Rightarrow ; 2m - frac{2}{3} lt k le 3m + frac{1}{2}tag{3}label{eq3}$$
      $$r = 2 ; Rightarrow ; 2m - frac{1}{3} lt k le 3m + 1 tag{4}label{eq4}$$
      $$r = 3 ; Rightarrow ; 2m lt k le 3m + frac{3}{2} tag{5}label{eq5}$$
      $$r = 4 ; Rightarrow ; 2m + frac{1}{3} lt k le 3m + 2 tag{6}label{eq6}$$
      $$r = 5 ; Rightarrow ; 2m + frac{2}{3} lt k le 3m + frac{5}{2} tag{7}label{eq7}$$



      With eqref{eq2} and eqref{eq3}, $k$ goes from $2m$ to $3m$ inclusive, so there are $m + 1$ values which work. Similarly with eqref{eq4}, $k$ goes from $2m$ to $3m + 1$, so there are actually $m + 2$ values which work. With eqref{eq5}, $k$ goes from $2m + 1$ to $3m + 1$, for $m + 1$ values. Finally, eqref{eq6} & eqref{eq7} give $k$ going from $2m + 1$ to $3m + 2$, for $m + 2$ values.



      As such, this works for all $n in mathbb{N}$, including for $n gt 9$. I'm not sure why this restriction is used in the question. Are there perhaps any other unstated conditions?






      share|cite|improve this answer











      $endgroup$
















        1












        1








        1





        $begingroup$

        Let $n = 6m + r$ for some non-negative integer $m$ and integer $0 le r le 5$. Thus, the question asks to prove there are more than $frac{n}{6} = m + frac{r}{6} lt m + 1$ values of $k$, i.e., that there are at least $m + 1$ such values.



        The inequality becomes



        $$2m + frac{r}{3} - 1 lt k le 3m + frac{r}{2} tag{1}label{eq1}$$



        In general for large divisors, you would normally look at various spans of values of $r$. However, since there aren't too many here, you can instead fairly easily manually check the results of the $6$ possible values of $r$ to get:



        $$r = 0 ; Rightarrow ; 2m - 1 lt k le 3m tag{2}label{eq2}$$
        $$r = 1 ; Rightarrow ; 2m - frac{2}{3} lt k le 3m + frac{1}{2}tag{3}label{eq3}$$
        $$r = 2 ; Rightarrow ; 2m - frac{1}{3} lt k le 3m + 1 tag{4}label{eq4}$$
        $$r = 3 ; Rightarrow ; 2m lt k le 3m + frac{3}{2} tag{5}label{eq5}$$
        $$r = 4 ; Rightarrow ; 2m + frac{1}{3} lt k le 3m + 2 tag{6}label{eq6}$$
        $$r = 5 ; Rightarrow ; 2m + frac{2}{3} lt k le 3m + frac{5}{2} tag{7}label{eq7}$$



        With eqref{eq2} and eqref{eq3}, $k$ goes from $2m$ to $3m$ inclusive, so there are $m + 1$ values which work. Similarly with eqref{eq4}, $k$ goes from $2m$ to $3m + 1$, so there are actually $m + 2$ values which work. With eqref{eq5}, $k$ goes from $2m + 1$ to $3m + 1$, for $m + 1$ values. Finally, eqref{eq6} & eqref{eq7} give $k$ going from $2m + 1$ to $3m + 2$, for $m + 2$ values.



        As such, this works for all $n in mathbb{N}$, including for $n gt 9$. I'm not sure why this restriction is used in the question. Are there perhaps any other unstated conditions?






        share|cite|improve this answer











        $endgroup$



        Let $n = 6m + r$ for some non-negative integer $m$ and integer $0 le r le 5$. Thus, the question asks to prove there are more than $frac{n}{6} = m + frac{r}{6} lt m + 1$ values of $k$, i.e., that there are at least $m + 1$ such values.



        The inequality becomes



        $$2m + frac{r}{3} - 1 lt k le 3m + frac{r}{2} tag{1}label{eq1}$$



        In general for large divisors, you would normally look at various spans of values of $r$. However, since there aren't too many here, you can instead fairly easily manually check the results of the $6$ possible values of $r$ to get:



        $$r = 0 ; Rightarrow ; 2m - 1 lt k le 3m tag{2}label{eq2}$$
        $$r = 1 ; Rightarrow ; 2m - frac{2}{3} lt k le 3m + frac{1}{2}tag{3}label{eq3}$$
        $$r = 2 ; Rightarrow ; 2m - frac{1}{3} lt k le 3m + 1 tag{4}label{eq4}$$
        $$r = 3 ; Rightarrow ; 2m lt k le 3m + frac{3}{2} tag{5}label{eq5}$$
        $$r = 4 ; Rightarrow ; 2m + frac{1}{3} lt k le 3m + 2 tag{6}label{eq6}$$
        $$r = 5 ; Rightarrow ; 2m + frac{2}{3} lt k le 3m + frac{5}{2} tag{7}label{eq7}$$



        With eqref{eq2} and eqref{eq3}, $k$ goes from $2m$ to $3m$ inclusive, so there are $m + 1$ values which work. Similarly with eqref{eq4}, $k$ goes from $2m$ to $3m + 1$, so there are actually $m + 2$ values which work. With eqref{eq5}, $k$ goes from $2m + 1$ to $3m + 1$, for $m + 1$ values. Finally, eqref{eq6} & eqref{eq7} give $k$ going from $2m + 1$ to $3m + 2$, for $m + 2$ values.



        As such, this works for all $n in mathbb{N}$, including for $n gt 9$. I'm not sure why this restriction is used in the question. Are there perhaps any other unstated conditions?







        share|cite|improve this answer














        share|cite|improve this answer



        share|cite|improve this answer








        edited Mar 20 at 3:10

























        answered Mar 20 at 2:24









        John OmielanJohn Omielan

        4,6312215




        4,6312215






























            draft saved

            draft discarded




















































            Thanks for contributing an answer to Mathematics Stack Exchange!


            • Please be sure to answer the question. Provide details and share your research!

            But avoid



            • Asking for help, clarification, or responding to other answers.

            • Making statements based on opinion; back them up with references or personal experience.


            Use MathJax to format equations. MathJax reference.


            To learn more, see our tips on writing great answers.




            draft saved


            draft discarded














            StackExchange.ready(
            function () {
            StackExchange.openid.initPostLogin('.new-post-login', 'https%3a%2f%2fmath.stackexchange.com%2fquestions%2f3154858%2ffind-the-number-of-k-satisfying-fracn3-1-k-leq-fracn2-for-n%23new-answer', 'question_page');
            }
            );

            Post as a guest















            Required, but never shown





















































            Required, but never shown














            Required, but never shown












            Required, but never shown







            Required, but never shown

































            Required, but never shown














            Required, but never shown












            Required, but never shown







            Required, but never shown







            Popular posts from this blog

            Magento 2 - Add success message with knockout Planned maintenance scheduled April 23, 2019 at 23:30 UTC (7:30pm US/Eastern) Announcing the arrival of Valued Associate #679: Cesar Manara Unicorn Meta Zoo #1: Why another podcast?Success / Error message on ajax request$.widget is not a function when loading a homepage after add custom jQuery on custom themeHow can bind jQuery to current document in Magento 2 When template load by ajaxRedirect page using plugin in Magento 2Magento 2 - Update quantity and totals of cart page without page reload?Magento 2: Quote data not loaded on knockout checkoutMagento 2 : I need to change add to cart success message after adding product into cart through pluginMagento 2.2.5 How to add additional products to cart from new checkout step?Magento 2 Add error/success message with knockoutCan't validate Post Code on checkout page

            Fil:Tokke komm.svg

            Where did Arya get these scars? Unicorn Meta Zoo #1: Why another podcast? Announcing the arrival of Valued Associate #679: Cesar Manara Favourite questions and answers from the 1st quarter of 2019Why did Arya refuse to end it?Has the pronunciation of Arya Stark's name changed?Has Arya forgiven people?Why did Arya Stark lose her vision?Why can Arya still use the faces?Has the Narrow Sea become narrower?Does Arya Stark know how to make poisons outside of the House of Black and White?Why did Nymeria leave Arya?Why did Arya not kill the Lannister soldiers she encountered in the Riverlands?What is the current canonical age of Sansa, Bran and Arya Stark?